Nilpotence et $\ker$

Salut

Soit $E$ un ev de dimension finie $n\in\mathbf N^*$ et $f\in\mathcal L(E)$ nilpotent. 1) et 2) sont équivalents :
1) L'indice de nilpotence de $f$ est $n$
2) $\dim\ker(f)=1$

1) $\implies$ 2) est facile mais je n'arrive pas 2) $\implies$ 1). Je connais la décroissance de $\big(\dim\ker(f^{k+1})-\dim\ker (f^k)\big)_{k\in\mathbf N}$ mais je ne sais pas comment l'utiliser ici. Si on note $p$ l'indice de nilpotence de $f$, je sais aussi que $p\leq n$.

Réponses

  • Petite erreur de signe : comme $\ker f^k\subset\ker f^{k+1}$, c'est la suite $\bigl(\dim\ker f^{k+1}-\dim\ker f^k\bigr)_{k\ge0}$ qui décroît.
    Avec l'hypothèse \[\dim\ker f^1-\dim\ker f^{0}=\dim\ker f=1,\] on a en notant $p$ l'indice de nilpotence : \[n=\dim\ker f^p=\sum_{k=1}^p\bigl(\dim\ker f^k-\dim\ker f^{k-1}\bigr)\le p.\]
  • Merci pour l'erreur de signe.

    D'où vient $\dim\ker f^p=\sum_{k=1}^p\bigl(\dim\ker f^k-\dim\ker f^{k-1}\bigr)$ ?
  • C'est une suite télescopique, pardi ! Est-ce que ça ne te saute pas aux yeux, en notant $d_k=\dim\ker f^k$ et en remarquant que $d_0=0$, que l'on a \[\sum_{k=1}^p(d_k-d_{k-1})=\sum_{k=1}^pd_k-\sum_{k=1}^pd_{k-1}=\sum_{k=1}^pd_k-\sum_{\ell=0}^{p-1}d_{\ell}=d_p-d_0=d_p\;?\]
    On peut aussi faire apparaître la suite des noyaux itérés et leurs quotients successifs, ce n'est jamais qu'une suite $(K_k)_{0\le k\le p}$ de sous-espaces emboîtés les uns dans les autres ($K_{k-1}\subset K_k$ pour $k\ge1$), ce qui donne : \begin{align*}
    \dim K_p&=\dim(K_p/K_{p-1})+\dim K_{p-1}\\
    \dim K_{p-1}&=\dim(K_{p-1}/K_{p-2})+\dim K_{p-2}\\
    \cdots&\\
    \dim K_1&=\dim(K_1/K_0)+\dim K_0.\end{align*}En ajoutant membre à membre et en simplifiant, on obtient l'égalité annoncée : \[\dim K_p
    =\sum_{k=1}^p\dim(K_k/K_{k-1})+\dim K_0=\sum_{k=1}^p\bigl(\dim K_k-\dim K_{k-1}\bigr)+\dim K_0.\]Avec ta remarque initiale que la suite des dimensions des quotients $K_k/K_{k-1}$ est décroissante et l'hypothèse que $\dim(K_1/K_0)=1$, on voit que chaque terme contribue au plus pour $1$ à la dimension $\dim K_p=n$ : il y a donc au moins $n$ termes, c'est-à-dire que $p\ge n$. C'est ce que j'ai écrit de façon moins explicite dans le message précédent.
  • Merci, c'est très clair.
Connectez-vous ou Inscrivez-vous pour répondre.